Re: Numbers in the Platonic Realm

2012-11-01 Thread Stephen P. King

On 11/1/2012 1:19 AM, meekerdb wrote:

On 10/31/2012 6:58 PM, Stephen P. King wrote:


Enumerate the programs computing functions fro N to N, (or the 
equivalent notion according to your chosen system). let us call those 
functions:  phi_0, phi_1, phi_2, ...  (the phi_i)

Let B be a fixed bijection from N x N to N. So B(x,y) is a number.

The number u is universal if phi_u(B(x,y)) = phi_x(y). And the 
equality means really that either both phi_u(B(x,y)) and phi_x(y) are 
defined (number) and that they are equal, OR they are both undefined.


In phi_u(B(x,y)) = phi_x(y), x is called the program, and y the data. 
u is the computer. u i said to emulate the program (machine, ...) x 
on the input y. 


So u could be any number, depending on how you enumerated the 
functions and what bijection is used?


Brent
--

Oh, BTW, Bruno wrote the above ... not me.

--
Onward!

Stephen

--
You received this message because you are subscribed to the Google Groups 
Everything List group.
To post to this group, send email to everything-list@googlegroups.com.
To unsubscribe from this group, send email to 
everything-list+unsubscr...@googlegroups.com.
For more options, visit this group at 
http://groups.google.com/group/everything-list?hl=en.



Re: Self-ascription and Perfect Model Model

2012-11-01 Thread Evgenii Rudnyi

On 30.10.2012 17:08 meekerdb said the following:

On 10/30/2012 4:45 AM, Evgenii Rudnyi wrote:


...


In this chapter, Van Fraassen has considered a map as a model for a
 typical model. A map is in the objective world, as well as a
scientific model, but to use the map one has to find out where on
the map he/she is located. I hope that you agree with that.


I don't agree with it because it's obviously false.  I just looked a
map to see how close Sandy came to my brother's home in Virginia.  I
didn't need to locate myself on that map.



In this case you need to locate your brother's home on that map. I do 
not see too much difference.


I would agree with you that my statement does not cover all possible 
cases that one could imagine to employ a map, but the act of location 
should be there anyway. There is a correspondence between a real world 
and a map but the map by itself does not coordinate the reality to 
itself. This is done by a human being.


Let me recall Van Fraassen's definition of a representation

p. 21 “Z uses X to depict Y as F”

The map seems to fit this pattern pretty well.

Could you imagine some case, when you use a map as a map and you do not 
need the act of location?


Evgenii
--
http://blog.rudnyi.ru/tag/bas-c-van-fraassen

--
You received this message because you are subscribed to the Google Groups 
Everything List group.
To post to this group, send email to everything-list@googlegroups.com.
To unsubscribe from this group, send email to 
everything-list+unsubscr...@googlegroups.com.
For more options, visit this group at 
http://groups.google.com/group/everything-list?hl=en.



Re: Self-ascription and Perfect Model Model

2012-11-01 Thread Evgenii Rudnyi

On 30.10.2012 16:25 Bruno Marchal said the following:


On 30 Oct 2012, at 12:53, Evgenii Rudnyi wrote:



...



You talk for example about integers as a framework for everything.
 Fine. Yet, I would like to understand how mankind through it
development has invented integers. How comp would help to answer
this?


Comp might not been able to answer that, in any better way than, say,
 evolution theory. Numbers are important in nature, as everything is
born from them, and to survive with bigger chance, the universal
numbers, us in particular, have to be able to recognize them, and
manipulate them accordingly. Comp is not a theory aimed at explaining
everything directly. It is just, at the start, an hypothesis in
philosophy of mind, and then it appears that it reduces the mind-body
problem to an explanation of quanta and qualia from
arithmetic/computer science.

Its main value in the human science, is, imo, that he forces us to be
 more modest, and more aware that we know about nothing, if only
because we have wrongly separate the human science (including
theology, afterlife, metaphysics)  and the exact sciences. Comp
provides a way to reunite them. Comp can be seen as an abstract
corpus callosum making a bridge between the formal and the informal,
before bridging mind and matter.


Below there is a couple of quotes about German idealism. Please replace 
Absolute Spirit by Natural Numbers there. Then it may give one possible 
answer to my question.


“Absolute Spirit is the fundamental reality. But in order to create the 
world, the Absolute manifests itself, or goes out of itself in a sense, 
the Absolute forgets itself and empties itself into creation (although 
never really ceasing to be itself). Thus the world is created as a 
“falling away” from Spirit, as a “self-alienation” of Spirit, although 
the Fall is never anything but a play of Spirit itself.”


“Having “fallen” into the manifest and material world, Spirit begins the 
process of returning to itself, and this process of the return of Spirit 
to Spirit is simply development or evolution itself. The original 
“descent” (or involution) is a forgetting, a fall, a self-alienation of 
Spirit; and the reverse movement of “ascent” (or evolution) is thus the 
self-remembering and self-actualization of Spirit. And yet, the 
Idealists emphasized, all of Spirit is fully present at each and every 
stage of evolution as the process of evolution itself. ”


Evgenii
--
http://blog.rudnyi.ru/2012/10/evolution-and-german-idealism.html

--
You received this message because you are subscribed to the Google Groups 
Everything List group.
To post to this group, send email to everything-list@googlegroups.com.
To unsubscribe from this group, send email to 
everything-list+unsubscr...@googlegroups.com.
For more options, visit this group at 
http://groups.google.com/group/everything-list?hl=en.



Re: Numbers in the Platonic Realm

2012-11-01 Thread Platonist Guitar Cowboy
On Thu, Nov 1, 2012 at 1:42 AM, Stephen P. King stephe...@charter.netwrote:

  On 10/31/2012 6:14 PM, Platonist Guitar Cowboy wrote:



 On Wed, Oct 31, 2012 at 7:59 PM, Stephen P. King stephe...@charter.netwrote:


  Dear Cowboy,

 One question. Was the general outline that I was trying to explain
 make any sense to you? Without being obvious about it, I am trying to
 finely parse the difference between the logic of temporal systems and the
 logic of atemporal systems - such as the Platonic Realm - such that I might
 show that reasonings that are correct in one are not necessarily correct in
 the other.


 This was not obvious to me, and going over the posts, I see how you're
 leaning that way... but why not just say that, then? Don't get me wrong, I
 love Joycean labyrinths as much as the next guy, but if the topic is on
 some level tending towards sincerity, then I don't see the benefit in not
 being obvious. Then again, I'm a Captain Obvious type. Should get the
 shirt.


 Hi Cowboy,

 I am dyslexic, this colors/flavors everything I write




 One problem that I have discovered (I thank Brent for bringing this up!)
 is that in our reasoning we set up constructions - such as the person on
 the desert island - that blur the very distinction that I am trying to
 frame. We should never assume temporal situations to argue for relations
 that are atemporal unless we are prepared to show the morphisms between the
 two situations.


 Isn't this already physical framework when you seem to be arguing for time
 as primitive (n incompatible with comp to begin with, after which you seek
 to carve out a distinction, when you've already mixed at the base?


 My argument is that it is impossible to 'derive Becoming from Being,
 but we can derive Being from Becoming. So why not work with the latter
 idea? I am trying to get Bruno to admit, among other things, that he has to
 assume a non-well founded logic for his result to work.;-)


I see less and less how you'd be able to do that, as I said, by making
process/linear time primitive in comp, and by assuming physical universe
with so many statements. Quantum Logic is part of the picture (see SANE
2004).





  Bruno would have us, in step 8 of UDA, to not assume a concrete
 robust physical universe. He goes on to argue that Occam's razor would
 demand that we reject the very idea of the existence of physical worlds
 given that he can 'show' how they can be reconstructed or derived from
 irreducible - and thus ontologically primitive - Arithmetic 'objects' {0,
 1, +, *} that are operating somehow in an atemporal way.


 UDA does not contradict itself here. Restraints on processing power, on
 memory and print capacities, implying time as some illusion emanating from
 eternal primitives, don't exist when framed non-constructively, more like
 sets of assignments, rather than operations in your sense, by which you
 seem to mean physically primitive operations on par with ontologically
 primitive arrow of time. Isn't this like cracking open the axioms, and then
 complaining that the building has cracks in it?


 There are simply a pile of concepts that are just assumed without
 explanation in any discussion of philosophy/logic/math. My point is that a
 theory must be have the capacity of being communicable ab initio for it to
 even be considered. When I am confronted with a theory or a result or an
 argument that seems to disallow for communicability I am going to baulk at
 it!


And the possibility that you are baulking at your preconceptions rather
than engaging the theory has never happened to you? Happens to me all the
time.






 We should be able to make the argument run without ever appealing to a
 Platonic realm or any kind of 'realism'.


 It's hard for me to see bets being made without some cash/investment/gap
 of faith on the table.


 Sure.


Then it would be easy for you to directly address the question: why assume
non-comp and then complain about comp's implications of time and physics
arising from dream interaction of universal numbers, therefore being not
primary or existing primitively?




 In my thinking, if arithmetic is powerful enough to be a TOE and run the
 TOE to generate our world, then that power should be obvious. My problem is
 that it looks tooo much like the 'explanation' of creation that we find in
 mythology, whether it is the 
 Ptahhttp://ancientegyptonline.co.uk/ptah.htmlof ancient Egypt or  the egg
 of Pangu http://www.livingmyths.com/Chinese.htm or whatever other myth
 one might like. What makes an explanation framed in the sophisticated and
 formal language of modal logic any different?


 Nothing, at its base. Appearances and looks can deceive, as numbers can
 too.


 Would this not make that deception something in our understanding and
 not the fault of numbers? After all, numbers are supposedly the least
 ambiguous of entities!


On the surface, but not when you look under the hood. That's a 

What is reality

2012-11-01 Thread Evgenii Rudnyi

A nice video

http://www.newscientist.com/video/1872152752001-what-is-reality.html

You have to ignore a short sponsor message at the beginning.

Evgenii

P.S. I have found it by

http://magpie73.livejournal.com/

--
You received this message because you are subscribed to the Google Groups 
Everything List group.
To post to this group, send email to everything-list@googlegroups.com.
To unsubscribe from this group, send email to 
everything-list+unsubscr...@googlegroups.com.
For more options, visit this group at 
http://groups.google.com/group/everything-list?hl=en.



Re: Numbers in the Platonic Realm

2012-11-01 Thread Stephen P. King

On 11/1/2012 6:54 AM, Platonist Guitar Cowboy wrote:



On Thu, Nov 1, 2012 at 1:42 AM, Stephen P. King stephe...@charter.net 
mailto:stephe...@charter.net wrote:


On 10/31/2012 6:14 PM, Platonist Guitar Cowboy wrote:



On Wed, Oct 31, 2012 at 7:59 PM, Stephen P. King
stephe...@charter.net mailto:stephe...@charter.net wrote:





  [SPK]

One problem that I have discovered (I thank Brent for
bringing this up!) is that in our reasoning we set up
constructions - such as the person on the desert island
- that blur the very distinction that I am trying to
frame. We should never assume temporal situations to
argue for relations that are atemporal unless we are
prepared to show the morphisms between the two situations.


Isn't this already physical framework when you seem to be
arguing for time as primitive (n incompatible with comp to
begin with, after which you seek to carve out a distinction,
when you've already mixed at the base?



My argument is that it is impossible to 'derive Becoming from
Being, but we can derive Being from Becoming. So why not work with
the latter idea? I am trying to get Bruno to admit, among other
things, that he has to assume a non-well founded logic for his
result to work.;-)


I see less and less how you'd be able to do that, as I said, by making 
process/linear time primitive in comp, and by assuming physical 
universe with so many statements. Quantum Logic is part of the picture 
(see SANE 2004).


Hi Cowboy,

I think of it this way: Change is fundamental (ala Heraclitus 
http://plato.stanford.edu/entries/heraclitus/#PhiPri and Bergson 
http://plato.stanford.edu/entries/bergson/#5) and Being is its 
automorphism http://en.wikipedia.org/wiki/Automorphism. Is that a bit 
more clear? Linear time (why 'linear'? Is there such a thing as 
non-linear time? Cyclic time is still linear, AFAIK...) is, IMHO, change 
+ a measure. Without a measure of change, there is no time; there is 
just change. If we take relativity seriously, we might even claim that 
there is no difference between change minus measure and staticness... I 
should mention that any change that has no measure associated with it is 
zeroth order change.
Without the means to compare two different things to each other, 
does it make any sense to be able to make coherent statements about some 
change in one relative to the other. If there is just one thing, how do 
we know anything about its possible change(s) unless we are looking at 
it and gauging (measuring) its change against some thing else that has 
some measure associated - but our observation of it violates the 
stipulation of if there is just one thing.


The idea that somehow the observer is irrelevant in physics and 
philosophy is, IMHO, one of the worse errors ever. Sure, we need to 
minimize and even eliminate observer bias and preferred reference 
framing, but eliminating the observer and replacing it with some 
ambiguous 'view from nowhere' is undiluted hogwash. This is where 
realist chafe me, they act as if the universe of objects is out there 
and has definite properties in the complete absence of any clear 
explanation for how those properties came to be defined in the first 
place. OK, OK, I will stop ranting...





Bruno would have us, in step 8 of UDA, to not assume a
concrete robust physical universe. He goes on to argue that
Occam's razor would demand that we reject the very idea of
the existence of physical worlds given that he can 'show' how
they can be reconstructed or derived from irreducible - and
thus ontologically primitive - Arithmetic 'objects' {0, 1, +,
*} that are operating somehow in an atemporal way.


UDA does not contradict itself here. Restraints on processing
power, on memory and print capacities, implying time as some
illusion emanating from eternal primitives, don't exist when
framed non-constructively, more like sets of assignments, rather
than operations in your sense, by which you seem to mean
physically primitive operations on par with ontologically
primitive arrow of time. Isn't this like cracking open the
axioms, and then complaining that the building has cracks in it?


There are simply a pile of concepts that are just assumed
without explanation in any discussion of philosophy/logic/math. My
point is that a theory must be have the capacity of being
communicable ab initio for it to even be considered. When I am
confronted with a theory or a result or an argument that seems
to disallow for communicability I am going to baulk at it!


And the possibility that you are baulking at your preconceptions 
rather than engaging the theory has never happened to you? Happens to 
me all the time.


OK, got any ideas what these might be other 

Re: Numbers in the Platonic Realm

2012-11-01 Thread Bruno Marchal


On 31 Oct 2012, at 19:59, Stephen P. King wrote:


On 10/30/2012 7:36 PM, Platonist Guitar Cowboy wrote:



On Tue, Oct 30, 2012 at 11:39 PM, Stephen P. King stephe...@charter.net 
 wrote:

On 10/30/2012 5:39 PM, meekerdb wrote:

On 10/30/2012 2:27 PM, Stephen P. King wrote:

On 10/30/2012 5:15 PM, meekerdb wrote:

On 10/30/2012 1:53 PM, Stephen P. King wrote:


 Dear Brent,

What is it that distinguishes between tokens and  
propositions?


Tokens are the physical elements (e.g. letters, words, sounds)  
that are used to represent a proposition in a particular language.


What determines the map between the letters, words, sounds  
and the content of propositions?


The proposition is the abstracted meaning which is independent  
of particular language.


Does this independence do so far as to disallow for an  
arbitrary physical entity to know of it? Independence of  
abstractions from particular individuals is not independence from  
all.


  So Zwei est ein und ein. are tokens expressing the same  
proposition as Two equals one plus one. which is that 2=1+1.


That


Which 'that' do you refer to, the tokens or the proposition.

is true only because multiple persons came to believe that it is  
true


You previously agreed that one person alone could come to know  
that 2=1+1 or 17 is prime and express it symbolically, i.e. in  
tokens.  So multiple persons are only necessary in order for the  
tokens to be used for communicating from one to another; which is  
the case whether the thing communicated is true or false.


  Reread this:


In 10/30/2012 5:03 PM, Stephen P. King wrote:

On 10/30/2012 3:05 PM, meekerdb wrote:
[SPK] Unless multiple entities can agree that the sequence of  
symbols 17 is prime is an indicator of some particular  
mathematical object and one of its particular properties, then  
how does 17 is prime come to mean anything at all?


I agree with that.  But you're talking about the tokens 17 is  
prime not the concept that 17 is prime.  Could not a person who  
grew up alone on an island realize that 17 has no divisors, and  
he could even invent a private language in which he could write  
down Peano's axioms.


Why are you using such trivial and parochial framing for  
abstract questions? Why the reference to single individuals? Did  
you not understand that I am claiming that meaningfulness requires  
at least the possibility of interaction between many entities such  
that each can evaluate the truth value of a proposition and thus  
can truthfully claim to have knowledge of true statements?
A person that grew and died on a desert island may have  
discovered for itself that 17 objects cannot be divided into equal  
subsets, but our statements about that are mere figemnts of our  
imagination as we could know nothing objective and non-imaginative  
at all about that person. We are imagining ourselves to have  
powers that we simply do not have. We are not omniscient voyeurs  
of Reality and there is not anything that is.


How is an imaginary entity come to aquire a real 1p or actual  
real properties? It might if that imaginary entity is deemed to  
have 1p content within some narrative. But outside of that  
narrative, it does not even exist! Languaging more about this is  
getting us nowhere.






Brent

and acted to cause it to be true. Remove one person from the  
multiplicity and the meaning still is there. Remove all of them  
and the meaning vanishes.






This needs a cowboy's few cents:

Every bet on ontological primitive is, despite the infinite models  
and conjectures we can weave from them, just that: a bet.


If this is stated clearly and honestly then it's cool, no matter if  
it turns out an error, as we've eliminated something at least.


But this is unfortunately rarer than to pound people with real,  
reality, authentic vs imaginary, artificial in discourse where  
axioms are not shared: if somebody can demarcate this boundary  
clearly for all discourse, then I fail to see/understand how  
anybody could do this outside of being high with a smile on their  
face and comic implication. My intelligence is limited insofar as I  
cannot understand, how this is not some form of needless force, in  
face of our vast ignorance.


Meaning is not some magical quality bestowed upon the discoverer of  
a set of relations. That's everybody's flavor of semantics working  
there.


As for human; if this is close to philosophical humanism  
semantically, then it's safe to say that, paired with standard  
model of physics, it's nice epistemologies with a lot of bs for its  
close association to ideological atheism; particularly the  
assertion no supernatural miracle shit when asserting singularity  
as big bang is just that: another miracle; when the rules of the  
humanist bet said no miracles.


m

Dear Cowboy,

One question. Was the general outline that I was trying to  
explain make any sense to you? Without being obvious about it, 

Re: Could universes in a multiverse be solipsistic ? Would this be a problem ?

2012-11-01 Thread Bruno Marchal


On 01 Nov 2012, at 00:35, Stephen P. King wrote:


On 10/31/2012 9:39 AM, Roger Clough wrote:

1) Yes, numbers float in a sea of universal mind (the One).

2) Here's a thought. If the universe acts like a gigantic
homunculus, with the supreme monad or One as its mind,
then could there be a solipsism to our universe such that
other multiverse versions of oiur universe could not access
(the mind of) ours ? Would this be a problem for multiverse
theories ?


Roger Clough,rclo...@verizon.net  10/31/2012

Dear Roger,

   I think that this idea is exactly wrong. The idea that numbers  
float in a sea of universal mind (the One) makes the explanation an  
infinite regress.


Replace the One by arithmetical truth, and the infinite regress  
disappear.


They reappear *in* arithmetical truth, but have fixed points (some  
provably, some non provably). No problem.






That is OK if and only if you allow for the concept of the One to be  
Kaufman and Zuckerman's Quine Atom aka Russell operator, but if not  
it does not work. Why? Because numbers have to be distinguishable  
from to have individual values. The totality of numbers is an  
infinity and thus have the property that their proper parts cannot  
be distinguished from their totality. How does the One accomplish  
this? It seems to me that we have to assume that the One is  
conscious of the numbers and that makes the numbers something  
different from the One for 1) to work and this is no different  
from what a finite mind does. My point here is that a mind cannot be  
infinite because it would be incapable of distinguishing it's self  
from any of its proper parts - making it the ultimate solipsist. Do  
there exist maps between the totality of an infinite set to an  
improper part? If yes, what are their necessary properties?


The One is solipsist, as the one is unique and alone. But I don't see  
why it should be conscious. It might be, but I see no evidence for this.


Bruno





   The idea of 2) seems to be demolished by Dennett's argument  
against the homunculus or else the One is strictly a solipsist as I  
argued above. I suspect that the mapping between wholes and improper  
parts is the same as Bruno's measure problem.


--
Onward!

Stephen


--
You received this message because you are subscribed to the Google  
Groups Everything List group.

To post to this group, send email to everything-list@googlegroups.com.
To unsubscribe from this group, send email to everything-list+unsubscr...@googlegroups.com 
.
For more options, visit this group at http://groups.google.com/group/everything-list?hl=en 
.




http://iridia.ulb.ac.be/~marchal/



--
You received this message because you are subscribed to the Google Groups 
Everything List group.
To post to this group, send email to everything-list@googlegroups.com.
To unsubscribe from this group, send email to 
everything-list+unsubscr...@googlegroups.com.
For more options, visit this group at 
http://groups.google.com/group/everything-list?hl=en.



Re: Numbers in the Platonic Realm

2012-11-01 Thread Bruno Marchal


On 01 Nov 2012, at 00:58, Stephen P. King wrote:


On 10/31/2012 12:22 PM, Bruno Marchal wrote:


On 30 Oct 2012, at 18:29, Stephen P. King wrote:


On 10/30/2012 12:38 PM, Bruno Marchal wrote:
No? If they do not have something equivalent to concepts, how  
can they dream?


Yes, the universal numbers can have concept.

Dear Bruno,

  Let's start over. Please plain in detail what is a universal  
number and how it (and not ordinary numbers) have concepts or 1p.


I will give more detail on FOAR, soon or later. But let me explains  
quickly.


Fix your favorite Turing universal system. It can be a programming  
language, a universal Turing machine, or a sigma_1 complete theory,  
or even a computer.


Dear Bruno,

   That 'fixing occurs at our level only. We are free (relatively)  
to fix our axiomatic objects from the wide variety that have been  
proven to exist within the Mathematical universe of concepts or, if  
we are clever, we can invent new concepts and work with them; but we  
cannot do things in our logic that are self-contradictory unless we  
make sure that the contradictions are not allowed to be pathological.


OK. No problem.







Enumerate the programs computing functions fro N to N, (or the  
equivalent notion according to your chosen system). let us call  
those functions:  phi_0, phi_1, phi_2, ...  (the phi_i)

Let B be a fixed bijection from N x N to N. So B(x,y) is a number.

The number u is universal if phi_u(B(x,y)) = phi_x(y). And the  
equality means really that either both phi_u(B(x,y)) and  phi_x(y)  
are defined (number) and that they are equal, OR they are both  
undefined.


In phi_u(B(x,y)) = phi_x(y), x is called the program, and y the  
data. u is the computer. u i said to emulate the program  
(machine, ...) x on the input y.




   OK, but this does not answer my question. What is the ontological  
level mechanism that distinguishes the u and the x and the y from  
each other?


The one you have chosen above. But let continue to use elementary  
arithmetic, as everyone learn it in school. So the answer is:  
elementary arithmetic.





What I am trying to explain to you that ontological level objects  
cannot have any logical mechanism that requires temporarily unless  
you are assuming some form of Becoming as an ontological primitive.  
Platonism, as far as I know, disallows this.


Indeed. becoming, like the whole physicalness, emerges from inside. It  
is 1p (plural).


Bruno








Comp is the thesis that I can survive with a physical digital  
computer in place of the physical brain, as far as it emulates me  
close enough.


Comp gives a special role to computer (physical incarnation of a  
universal number). The comp idea is that computer can supports  
thinking and consciousness, and makes them capable of manifestation  
relatively to other universal structure (physical universes if that  
exists, people, etc.). This should answer your question.


The lobian machines are only universal numbers, having the  
knowledge that they are universal.
I can prove to any patient human that he/she is Löbian (I cannot  
prove that he/she is sound or correct, note).


The UDA results is that whatever you mean by physical for making  
comp meaningful, that physicalness has to emerge entirely and only,  
from a 'competition' between all universal numbers. There is no  
need to go out of arithmetic, and worst, there is no possible use  
of going out of arithmetic, once betting on comp.


By arithmetic I mean arithmetical truth, or the standard model of  
arithmetic, I don't mean a theory. I mean the whole set of true  
arithmetical propositions, or of their Gödel numbers.


Bruno





--
Onward!

Stephen


--
You received this message because you are subscribed to the Google  
Groups Everything List group.

To post to this group, send email to everything-list@googlegroups.com.
To unsubscribe from this group, send email to everything-list+unsubscr...@googlegroups.com 
.
For more options, visit this group at http://groups.google.com/group/everything-list?hl=en 
.




http://iridia.ulb.ac.be/~marchal/



--
You received this message because you are subscribed to the Google Groups 
Everything List group.
To post to this group, send email to everything-list@googlegroups.com.
To unsubscribe from this group, send email to 
everything-list+unsubscr...@googlegroups.com.
For more options, visit this group at 
http://groups.google.com/group/everything-list?hl=en.



Re: Numbers in the Platonic Realm

2012-11-01 Thread Bruno Marchal


On 01 Nov 2012, at 01:18, Stephen P. King wrote:


On 10/31/2012 12:45 PM, Bruno Marchal wrote:


On 30 Oct 2012, at 18:39, Stephen P. King wrote:


On 10/30/2012 12:51 PM, Bruno Marchal wrote:


On 30 Oct 2012, at 17:04, meekerdb wrote:


On 10/30/2012 4:30 AM, Bruno Marchal wrote:


My argument is that concepts of truth and provability of  
theorems apply only to the concepts of numbers and their  
constructions, not to numbers themselves.


Truth applies to proposition, or sentences representing them  
for some machine/numbers. If not, comp does not even makes sense.


So your are agreeing?  Two has no truth value, but Two equals  
one plus one. does.


Yes I agree. It seems I insisted on this a lot.
But in this context, it seems that Stephen was using this to  
assert that the truth of, say  Two equals one plus one. depend  
on some numbers or subject having to discover it, or prove it.


Bruno

http://iridia.ulb.ac.be/~marchal/



Dear Bruno,

My point is that a number is not a capable of being an  
ontological primitive


Then I can stop reading as you need to assume the numbers (or  
anything Turing equivalent) to get them.


Dear Bruno,

So it is OK to assume that which I seek to explain?



You can't explain the numbers without assuming the numbers. This has  
been foreseen by Dedekind, and vert well justified by many theorem in  
mathematical logic. Below the number, you are lead to version of  
ultrafinitism, which is senseless in the comp theory.








*and* having some particular set of values and meanings.


I just assume

x + 0 = x
x + s(y) = s(x + y)

 x *0 = 0
 x*s(y) = x*y + x

And hope you understand.



I can understand these symbols because there is at least a way  
to physically implement them.


Those notion have nothing to do with physical implementation.
Implementation and physical will be explained from them. A natural  
thing as they are much more complex than the laws above.






In the absence of some common media, even if it is generated by  
sheaves of computations, there simply isno way to understand  
anything.


Why ?




You must accept non-well foundedness for your result to work, but  
you seem fixated against that.


1004.







A statement, such as 2 = 1+1 or two equals one plus one, are said  
truthfully to have the same meaning because there are multiple and  
separable entities that can have the agreement on the truth value.  
In the absence of the ability to judge a statement independently  
of any particular entity capable of understanding the statement,  
there is no meaning to the concept that the statement is true or  
false. To insist that a statement has a meaning and is true (or  
false) in an ontological condition where no entities capable of  
judging the meaning, begs the question of meaningfulness!
   You are taking for granted some things that your arguments  
disallow.



Do you agree that during the five seconds just after the Big Bang  
(assuming that theory) there might not have been any possible  
observers. But then the Big Bang has no more sense.


No, I don't. Why? Because that concept of the five seconds just  
after the Big Bang is an assumption of a special case or pleading.  
I might as well postulate the existence of Raindow Dash to act as  
the entity to whom the Truth of mathematical statements have  
absolute meaning. To be frank, I thing that the Big Bang theory, as  
usually explained is a steaming pile of rubbish, as it asks us to  
believe that the totality of all that exists sprang into being from  
Nothing.


I actually agree, by accident, on this. But this is not relevant for  
my point. Imagine that we can show that some solution to GR equantion  
have universe so poor that life cannot exist in there, would you say  
that such universe cannot exist?




I believe that the totality of what exists is eternal, having no  
beginning and no end.


I am OK with that. It is close to Platonism. But with comp we can  
restrict this to the arithmetical truth (a highly non computable  
structure, but still conceivable by universal numbers, relatively).




What we infer from our observations of Hubble expansion is just an  
effect that follows, ultimately, from our finiteness.


Including time and space. So we do agree again.







I think Brent is right, and Quentin. You confuse 1+1=2 with human  
expression for pointing on that proposition. You obviously needs  
human to understand those  1+1=2 , but the content of 1+1=2  
has simply no relation at all with the human, or with a physical  
universe.


No, none of you have yet to be able to understand my counter- 
argument. It is not complicated. We cannot assume to have something  
when the means for its existence is not allowed. My claim is that  
meaningfulness supervenes on the possibility of interaction of  
*many* entities and is independent of any *one* (or some lesser  
finite subset) of that Many.


But arithmetical truth is full of entities, even 

Re: Numbers in the Platonic Realm

2012-11-01 Thread Bruno Marchal


On 01 Nov 2012, at 05:27, meekerdb wrote:


On 10/31/2012 11:52 AM, Bruno Marchal wrote:


I don't see why denying mathematical realism would entail saying  
no to the doctor.


It implies not saying yes qua computatio. It implies NOT  
understanding what Church thesis is about, as to show it consistent  
you need the diagonalization, which use the excluded middle  
principle.

You can still say yes, but only by using some magic.



The doctor isn't proposing to replace part of you brain with a  
piece of Platonia, he has a real physical device to implant.


This is not related. That will follow step 8.

Here, you have to be arithmetical realist to get an idea of what a  
computer is, and how it functions, as the physical one will  
approximate it, well enough, it is hoped.


Of course you can say yes to the doctor, just because you trust  
him. But comp is not saying yes to the doctor. Comp is the  
doctrine that saying yes will indeed work, once the artificial  
brain is a *computer*. The definition of computer makes no sense  
with arithmetical realism.


?? If I'm a materialist I could say yes because I think the  
artificial brain produces the same input/output signals.


But you need to be arithmetical realist to define what you mean by  
same input-output.
Arithmetical realism is not a big deal. It means that you believe that  
2+2=5 OR 2+2≠5.







  I don't see why I would have believe in Platonia.


Comp use only arithmetical platonia, and that is just a poetical  
expression to say that you believe that 17 is prime independently of  
the existence of the Higgs boson.



I may believe that only some computations are instantiated and there  
are no infinities.


OK, but again, that is different. That's the move toward physical  
ultrafinitism. You can keep comp, up to step seven, and we are back on  
the fact that step 8 (the movie-graph argument) makes such move  
senseless.


But note that to just define the term computation, you need to be  
arithmetical realist. But if there were no step 8, indeed, you might  
have succeed in saving a form of materialism.  I still miss what you  
don't understand in the step 8. you did not comment my recent answer  
on this.
Maybe you could try to elaborate on your intuition. Why and how does a  
primitive matter change something in a computation or in the  
consciousness associated to it, and this in a Turing emulable manner?


Bruno


http://iridia.ulb.ac.be/~marchal/



--
You received this message because you are subscribed to the Google Groups 
Everything List group.
To post to this group, send email to everything-list@googlegroups.com.
To unsubscribe from this group, send email to 
everything-list+unsubscr...@googlegroups.com.
For more options, visit this group at 
http://groups.google.com/group/everything-list?hl=en.



Re: Numbers in the Platonic Realm

2012-11-01 Thread Bruno Marchal


On 01 Nov 2012, at 06:19, meekerdb wrote:


On 10/31/2012 6:58 PM, Stephen P. King wrote:  (actually it was Bruno)



Enumerate the programs computing functions fro N to N, (or the  
equivalent notion according to your chosen system). let us call  
those functions:  phi_0, phi_1, phi_2, ...  (the phi_i)

Let B be a fixed bijection from N x N to N. So B(x,y) is a number.

The number u is universal if phi_u(B(x,y)) = phi_x(y). And the  
equality means really that either both phi_u(B(x,y)) and  phi_x(y)  
are defined (number) and that they are equal, OR they are both  
undefined.


In phi_u(B(x,y)) = phi_x(y), x is called the program, and y the  
data. u is the computer. u i said to emulate the program  
(machine, ...) x on the input y.


So u could be any number, depending on how you enumerated the  
functions and what bijection is used?


Any number. I am not sure, the enumeration has to be given by an  
algorithm.


But yes, the notion of computation, universality, etc. are intensional  
notion, and makes sense only relatively to the other number. That is  
why a often add relative before number.


This should be obvious. The doctor who scan your brain will also have  
some flexibility in the encoding of your current local and relative  
state.

I doubt it can encode it with the number 4, though.
You might say, that 4 is for the fourth compact disk on the shell  
doctor, but then 4 is no more an encoding, but only a pointer to an  
encoding.


Bruno



http://iridia.ulb.ac.be/~marchal/



--
You received this message because you are subscribed to the Google Groups 
Everything List group.
To post to this group, send email to everything-list@googlegroups.com.
To unsubscribe from this group, send email to 
everything-list+unsubscr...@googlegroups.com.
For more options, visit this group at 
http://groups.google.com/group/everything-list?hl=en.



Re: Self-ascription and Perfect Model Model

2012-11-01 Thread meekerdb

On 11/1/2012 5:03 AM, Evgenii Rudnyi wrote:

On 30.10.2012 17:08 meekerdb said the following:

On 10/30/2012 4:45 AM, Evgenii Rudnyi wrote:


...


In this chapter, Van Fraassen has considered a map as a model for a
typical model. A map is in the objective world, as well as a
scientific model, but to use the map one has to find out where on
the map he/she is located. I hope that you agree with that.


I don't agree with it because it's obviously false. I just looked a
map to see how close Sandy came to my brother's home in Virginia. I
didn't need to locate myself on that map.



In this case you need to locate your brother's home on that map. I do not see too much 
difference.


I would agree with you that my statement does not cover all possible cases that one 
could imagine to employ a map, but the act of location should be there anyway. There is 
a correspondence between a real world and a map but the map by itself does not 
coordinate the reality to itself. This is done by a human being.


Let me recall Van Fraassen's definition of a representation

p. 21 “Z uses X to depict Y as F”

The map seems to fit this pattern pretty well.

Could you imagine some case, when you use a map as a map and you do not need the act of 
location?


Of course not. But in many cases I do not need to locate myself, which was crucial to van 
Frassen's point about self reference.


Brent

--
You received this message because you are subscribed to the Google Groups 
Everything List group.
To post to this group, send email to everything-list@googlegroups.com.
To unsubscribe from this group, send email to 
everything-list+unsubscr...@googlegroups.com.
For more options, visit this group at 
http://groups.google.com/group/everything-list?hl=en.



Re: Self-ascription and Perfect Model Model

2012-11-01 Thread Evgenii Rudnyi

On 01.11.2012 18:00 meekerdb said the following:

On 11/1/2012 5:03 AM, Evgenii Rudnyi wrote:

On 30.10.2012 17:08 meekerdb said the following:

On 10/30/2012 4:45 AM, Evgenii Rudnyi wrote:


...


In this chapter, Van Fraassen has considered a map as a model
for a typical model. A map is in the objective world, as well
as a scientific model, but to use the map one has to find out
where on the map he/she is located. I hope that you agree with
that.


I don't agree with it because it's obviously false. I just looked
a map to see how close Sandy came to my brother's home in
Virginia. I didn't need to locate myself on that map.



In this case you need to locate your brother's home on that map. I
do not see too much difference.

I would agree with you that my statement does not cover all
possible cases that one could imagine to employ a map, but the act
of location should be there anyway. There is a correspondence
between a real world and a map but the map by itself does not
coordinate the reality to itself. This is done by a human being.

Let me recall Van Fraassen's definition of a representation

p. 21 “Z uses X to depict Y as F”

The map seems to fit this pattern pretty well.

Could you imagine some case, when you use a map as a map and you do
 not need the act of location?


Of course not. But in many cases I do not need to locate myself,
which was crucial to van Frassen's point about self reference.



It was just a special case. It was my fault that I have not described it 
better. In the general case this is The Problem of Coordination (chapter 
5 in the book).


Evgenii

--
You received this message because you are subscribed to the Google Groups 
Everything List group.
To post to this group, send email to everything-list@googlegroups.com.
To unsubscribe from this group, send email to 
everything-list+unsubscr...@googlegroups.com.
For more options, visit this group at 
http://groups.google.com/group/everything-list?hl=en.



Re: Self-ascription and Perfect Model Model

2012-11-01 Thread Bruno Marchal


On 01 Nov 2012, at 11:09, Evgenii Rudnyi wrote:


On 30.10.2012 16:25 Bruno Marchal said the following:


On 30 Oct 2012, at 12:53, Evgenii Rudnyi wrote:



...



You talk for example about integers as a framework for everything.
Fine. Yet, I would like to understand how mankind through it
development has invented integers. How comp would help to answer
this?


Comp might not been able to answer that, in any better way than, say,
evolution theory. Numbers are important in nature, as everything is
born from them, and to survive with bigger chance, the universal
numbers, us in particular, have to be able to recognize them, and
manipulate them accordingly. Comp is not a theory aimed at explaining
everything directly. It is just, at the start, an hypothesis in
philosophy of mind, and then it appears that it reduces the mind-body
problem to an explanation of quanta and qualia from
arithmetic/computer science.

Its main value in the human science, is, imo, that he forces us to be
more modest, and more aware that we know about nothing, if only
because we have wrongly separate the human science (including
theology, afterlife, metaphysics)  and the exact sciences. Comp
provides a way to reunite them. Comp can be seen as an abstract
corpus callosum making a bridge between the formal and the informal,
before bridging mind and matter.


Below there is a couple of quotes about German idealism. Please  
replace Absolute Spirit by Natural Numbers there.


Here there is a little difficulty. With comp, the absolut Spirit (or  
the one, or God, or the big thing without a name, etc...) can be the  
Natural Numbers (I would add structured by + and *, as it is not just  
0, 1, 2, ...).


But then comp can prove that there is no way the creatures itself  
could ever know that, nor even completely able to explain what that  
could mean.


There is no paradox. There is only a remind that we cannot prove that  
comp is true.


If ever we could prove comp, then we could give a name to God, and  
then we fall ... again.





Then it may give one possible answer to my question.

“Absolute Spirit is the fundamental reality. But in order to create  
the world, the Absolute manifests itself, or goes out of itself in a  
sense, the Absolute forgets itself and empties itself into creation  
(although never really ceasing to be itself). Thus the world is  
created as a “falling away” from Spirit, as a “self-alienation” of  
Spirit, although the Fall is never anything but a play of Spirit  
itself.”


Yes. This remind me of my quoting of Aurobindo, which suits so well  
something in both comp, and, swim has to say, the salvia divinorum  
experience:



What, you ask, was the beginning of it all?

And it is this ...
Existence that multiplied itself
For sheer delight of being
And plunged into numberless trillions of forms
So that it might
Find
Itself
Innumerably (Aurobindo)






“Having “fallen” into the manifest and material world, Spirit begins  
the process of returning to itself, and this process of the return  
of Spirit to Spirit is simply development or evolution itself. The  
original “descent” (or involution) is a forgetting, a fall, a self- 
alienation of Spirit; and the reverse movement of “ascent” (or  
evolution) is thus the self-remembering and self-actualization of  
Spirit. And yet, the Idealists emphasized, all of Spirit is fully  
present at each and every stage of evolution as the process of  
evolution itself. ”


I can't agree more, Evgenii.

It is also very close to Plotinus' emanation/conversion, which has  
influenced the Christians a lot, even if this has often taken the  
shape of fairy tales (which obviously should never been taken  
literally).


In Plotinus, and in comp we can say more: it is the process of  
conversion of the Soul toward the Spirit, which literally create the  
material reality. It is the indeterminateness of the border of the  
universal mind, where God loss control, so to speak, which makes it  
possible for the soul to start the conversion, and come back to the  
source. That conception of matter is already in Aristotle, but  
Plotinus gives the Platonist correction which makes it consistent, and  
even necessary I would say, with the theology, including physics, of  
the universal machines.
Each individual universal machines is a window for the arithmetical  
truth to discover (partially) itself, but also losing itself, in  
itself. This entails  a double amnesia: God has to forget his identity  
to explore itself, and the creatures have to forget the window and the  
exploration  (the body and the environment) to remember who they are.
Likewise with salvia: you forget who you are here to remind who you  
are there, and vice versa, apparently from many reports, although you  
can also just disconnect, instead of forgetting (which is handy when  
the phone rings).


Indeed, the same with comp once you accept the greek definition of  
knowledge and dream---roughly speaking: true belief 

Re: Numbers in the Platonic Realm

2012-11-01 Thread Bruno Marchal


On 01 Nov 2012, at 14:25, Stephen P. King wrote:


   But I agree with comp up to the strong version of step 8!



But then you have to find the flaw in step 8. as step 8 is done in  
comp, without adding any assumptions, of course.




I accept comp with a weak version of step 8 or, I think  
equivalently, a weak version of computational universality: A  
computation is universal if it is not dependent on any one  
particular physical system.


This is called functional, not universal. It has nothing to do with  
Turing universality.




This implies, to me, that there is at least one physical system that  
such a universal computation can be said to actually run on!


I don't see this.





This goes against the Parmenidean/Platonistic idea of computation as  
static objects in eternity that are completely independent of  
physical stuff!


Sorry but, by definition, computations are static objects in  
arithmetic (or in fortanic, Lispic, combinatoric, lambdaic, etc  
There are a lot of equivalent ontological choices here.).


The physicist have not (yet) found a definition of computation which  
does not use that mathematical definition. This exists, though, has  
*many* physical systems are in principle Turing universal. But Turing  
universal is a mathematical, even arithmetical, (even in the strong  
logician sense).


Bruno



http://iridia.ulb.ac.be/~marchal/



--
You received this message because you are subscribed to the Google Groups 
Everything List group.
To post to this group, send email to everything-list@googlegroups.com.
To unsubscribe from this group, send email to 
everything-list+unsubscr...@googlegroups.com.
For more options, visit this group at 
http://groups.google.com/group/everything-list?hl=en.



Re: Self-ascription and Perfect Model Model

2012-11-01 Thread Evgenii Rudnyi

On 01.11.2012 18:30 Bruno Marchal said the following:


On 01 Nov 2012, at 11:09, Evgenii Rudnyi wrote:



...


“Absolute Spirit is the fundamental reality. But in order to create
 the world, the Absolute manifests itself, or goes out of itself in
a sense, the Absolute forgets itself and empties itself into
creation (although never really ceasing to be itself). Thus the
world is created as a “falling away” from Spirit, as a
“self-alienation” of Spirit, although the Fall is never anything
but a play of Spirit itself.”


Yes. This remind me of my quoting of Aurobindo, which suits so well
something in both comp, and, swim has to say, the salvia divinorum
experience:

 What, you ask, was the beginning of it all?

And it is this ... Existence that multiplied itself For sheer delight
of being And plunged into numberless trillions of forms So that it
might Find Itself Innumerably (Aurobindo)








“Having “fallen” into the manifest and material world, Spirit
begins the process of returning to itself, and this process of the
return of Spirit to Spirit is simply development or evolution
itself. The original “descent” (or involution) is a forgetting, a
fall, a self-alienation of Spirit; and the reverse movement of
“ascent” (or evolution) is thus the self-remembering and
self-actualization of Spirit. And yet, the Idealists emphasized,
all of Spirit is fully present at each and every stage of evolution
as the process of evolution itself. ”


I can't agree more, Evgenii.

It is also very close to Plotinus' emanation/conversion, which has
influenced the Christians a lot, even if this has often taken the
shape of fairy tales (which obviously should never been taken
literally).

In Plotinus, and in comp we can say more: it is the process of
conversion of the Soul toward the Spirit, which literally create the
 material reality. It is the indeterminateness of the border of the
universal mind, where God loss control, so to speak, which makes it
possible for the soul to start the conversion, and come back to the
source. That conception of matter is already in Aristotle, but
Plotinus gives the Platonist correction which makes it consistent,
and even necessary I would say, with the theology, including physics,
of the universal machines. Each individual universal machines is a
window for the arithmetical truth to discover (partially) itself, but
also losing itself, in itself. This entails  a double amnesia: God
has to forget his identity to explore itself, and the creatures have
to forget the window and the exploration  (the body and the
environment) to remember who they are. Likewise with salvia: you
forget who you are here to remind who you are there, and vice versa,
apparently from many reports, although you can also just disconnect,
instead of forgetting (which is handy when the phone rings).

Indeed, the same with comp once you accept the greek definition of
knowledge and dream---roughly speaking: true belief (knowledge) and
consistent belief (dream).

Bruno


http://iridia.ulb.ac.be/~marchal/




Then you may like

Ken Wilber, The Marriage of Sense and Soul: Integrating Science and Religion

as the quotes have been from this book. There is nothing about comp 
there but the book is not that bad.


I should say that the author understands the religion pretty general: as 
spirit and contemplation. It is an interesting overview  of how science 
has started to dominate over art and moral and what could be done 
against it. The different would-be solutions from history are also 
considered. Wilber’s solution, if I have understood correctly, goes like 
a combination of German idealism (that you like) + Joga.


Evgenii
--

http://blog.rudnyi.ru/2012/10/the-marriage-of-sense-and-soul.html


--
You received this message because you are subscribed to the Google Groups 
Everything List group.
To post to this group, send email to everything-list@googlegroups.com.
To unsubscribe from this group, send email to 
everything-list+unsubscr...@googlegroups.com.
For more options, visit this group at 
http://groups.google.com/group/everything-list?hl=en.



Re: Numbers in the Platonic Realm

2012-11-01 Thread Stephen P. King

On 11/1/2012 11:23 AM, Bruno Marchal wrote:
[SPK] Bruno would have us, in step 8 of UDA, to not assume a 
concrete robust physical universe.


?

Reread step 8. Step 7 and step 8 are the only steps where I explicitly 
do assume a primitive physical reality.

In step 8, it is done for the reductio ad absurdum.


Dear Bruno,

   I have cut and pasted your exact words from SANE04 and you still 
didn't understand... From: 
http://iridia.ulb.ac.be/~marchal/publications/SANE2004MARCHAL.pdf 
http://iridia.ulb.ac.be/%7Emarchal/publications/SANE2004MARCHAL.pdf


...what  if we  don't  grant a concrete robust  physical universe?
Actually the 8th present step will  explain
that such a move is nevertheless without purpose. This will make the 
notion of concrete and
existing universe completely devoid of  any  explicative  power. It  
will  follow  that  a  much
weaker and usual form of Ockham's razor can be used to conclude that not 
only physics has
been  epistemologically reduced  to  machine  psychology, but that 
''matter'' has  been
ontologically reduced to ''mind'' where mind is defined  as the object 
study of fundamental

machine psychology.

My claim is that _/*neither physical worlds nor numbers (or any 
other object that must supervene on mind) can be ontologically 
primitive*/_. Both must emerge from a neutral ground that is neither and 
has no particular properties.





[SPK] He goes on to argue that Occam's razor would demand that we 
reject the very idea of the existence of physical worlds


Only of primitive physical worlds. And you did agree with this. I just 
prove this from comp. That's the originality. A bit of metaphysics is 
made into a theorem in a theory (comp).


Can we agree that physical worlds emerge somehow from sharable 
aspects of multiple sheaves of computations?





[SPK]  given that he can 'show' how they can be reconstructed or 
derived from irreducible - and thus ontologically primitive - 
Arithmetic 'objects' {0, 1, +, *} that are operating somehow in an 
atemporal way. We should be able to make the argument run without 
ever appealing to a Platonic realm or any kind of 'realism'. In my 
thinking, if arithmetic is powerful enough to be a TOE and run the 
TOE to generate our world, then that power should be obvious. My 
problem is that it looks tooo much like the 'explanation' of creation 
that we find in mythology, whether it is the Ptah 
http://ancientegyptonline.co.uk/ptah.html of ancient Egypt or  the 
egg of Pangu http://www.livingmyths.com/Chinese.htm or whatever 
other myth one might like. What makes an explanation framed in the 
sophisticated and formal language of modal logic any different?


I use the self-reference logic, for obvious reason. Again, this 
entails the sue of some modal logics, due to a *theorem* by Solovay. 
All correct machine whose beliefs extend RA obeys to G and G*. There 
is no choice in the matter.


That is not changed or involved by my argument.





[SPK] I agree 10% with your point about 'miracles'. I am 
very suspicions of special explanations' or 'natural conspiracies'.  
(This comes from my upbringing as a Bible-believing Fundamentalist 
and eventual rejection of that literalist mental straight-jacket.) As 
I see things, any condition or situation that can be used to 
'explain' some other conceptually difficult condition or situation 
should be either universal in that they apply anywhere and anytime


But even in your theory anywhere and anytime must be defined by 
something more primitive, given that you agree that physics cannot be 
the fundamental theory, given that the physical reality is not primitive.


The concepts of where and when (positions in a space-time) 
would seem to be rendered meaningless if there is no space-time (or 
observers/measurements to define it), no? OH, BTW, I don't think that we 
disagree that physics cannot be the fundamental theory. Physics 
requires measurements/observations to be meaningful. Where I agree with 
you is in your considerations of 1p and observer indeterminacy. Where 
you and I disagree is on the question of resources. Resources are 
required for computations to run so there has to be the availability 
of resources involved in *any* consideration of computations. Ignoring 
these considerations by only considering computations as Platonic 
objects is wrong, IMHO.
You seem to be OK with computations as purely timeless objects (in 
Platonia) that are such that somehow we finite entities can create 
physical objects that can implement (in their dynamical functions) 
instances of such, while I claim that computations are equivalence 
classes of functions that physical systems can implement *and* abstract 
objects. I see these two views as two poles of a spectrum. There is a 
lot more detail in my considerations that I do not have time to go into 
at this time...


My Theory of comp: Sheaves of Computations/arithmetic - define - 
particular physical states 

Re: Against Mechanism

2012-11-01 Thread John Clark
On Wed, Oct 31, 2012 at 2:21 PM, Bruno Marchal marc...@ulb.ac.be wrote:

 the you before the duplication or the you after the duplication?


  All the you after, are the you before, by definition of comp.


OK, but the you before is not the you after. The Helsinki man knows nothing
about Moscow or Washington, not even if he still exists after the
duplication, but both the Moscow man and the Washington man know all about
Helsinki even if they don't know about each other.

 what you will live, as a first person.


If your mind works deterministically then what you will live to think you
see will depend on the external environment. If your mind does NOT work
deterministically then what you will live to think you see will depend on
absolutely nothing, in other words it is random. There is no new sort of
indeterminacy involved just the boring old sort, and how you expect to draw
profound philosophical  conclusions from such a flimsy foundation is a
mystery.

 You know by comp that [...]


I don't know anything by comp. At one time I thought I knew what you
meant by the term, but then you say consciousness was there before
Evolution produced brains and that the owner [of a brain] itself must
attach his consciousness to all states existing in arithmetic. So I was
wrong, I don't know what comp means.

 Before the duplication the you is the Helsinki man, after the
 duplication the you is the Helsinki man and the Washington man and the
 Moscow man. What is the probability the Helsinki man will write in his
 diary that he sees Washington? 0%.


  The guy reconstituted in Washington will say: Gosh I was wrong.


That's the problem, you're not clear who I is. The Washington man made no
error because he made no predictions of any sort, only the Helsinki man did
that. The Washington man and Helsinki man have identical memories up to the
point of duplication but after that they diverge.

  What is the probability the Helsinki man will write in his diary he
 sees Helsinki? 100%.

  No. In the protocol that I have described to you many times, the
 probability here is 0%, as he is cut and pasted. Not copy and pasted.


If the Helsinki man had never seen Helsinki then he's not the Helsinki man,
if he has seen that city then he wrote so in his diary.

 And it is not he sees but what will he see. And the protocol assures
 that he will only see washington, or Moscow.


Who is he?

  What is the probability the Washington man will write in his diary he
 sees Washington? 100%.

  The question was asked to the Helsinki man.


But you said the Helsinki man was destroyed, if so then he's got a rather
severe case of writers block and is writing very little in his diary.

   And if the duplicating process destroys the Helsinki man then the
 probability the Helsinki man will write anything at all in his diary is 0%.

  Then comp is false.


OK if you say so, its your invention so whatever comp means its false;
although I am a little surprised that you expect a man who no longer exists
to write stuff in his diary.

 The question is about your first person experience. [...]The question is
 not about you, but about the most probable result of an experiment that you
 can do. You push on a button, and you localize your directly accessible
 body.


Your? You? John Clark believes that when considering matters of identity if
Bruno Marchal stopped using so many pronouns without considering what they
refer to then Bruno Marchal's thinking would be less muddled.

As Quentin said, it is implicit in the Everett understanding of QM.


In Everett a world does not split until there is a difference between them
and neither does consciousness. And the same is true in the thought
experiment, If Bruno Marchal's body is duplicated and sent to Washington
and Moscow but inside identical boxes then Bruno Marchal's consciousness
has not been duplicated and will not be until the boxes are opened and
different things are observed by the Brunos, at that point they will no
longer be each other but both will still beBruno Marchal

 In most physics experiments, even very advanced ones at CERN, the
 experimenter himself is not duplicated so in the question What particle do
 you expect to see? it's clear who you is;


  Only if you assume that the universe does not contain Boltzman brains,
 or a universal dovetailer,


It doesn't matter if Boltzman brains exist or not. In physics experiments
not involving self duplications which you is involved is obvious, and it
can be proven to be correct by observing that when you predicts what
you will see using physical laws the prediction usually proves to be
true, so all the yous must have been assigned correctly.

  John K Clark

-- 
You received this message because you are subscribed to the Google Groups 
Everything List group.
To post to this group, send email to everything-list@googlegroups.com.
To unsubscribe from this group, send email to 
everything-list+unsubscr...@googlegroups.com.
For more options, visit this 

Re: Could universes in a multiverse be solipsistic ? Would this be a problem ?

2012-11-01 Thread Stephen P. King

On 11/1/2012 11:36 AM, Bruno Marchal wrote:


On 01 Nov 2012, at 00:35, Stephen P. King wrote:


On 10/31/2012 9:39 AM, Roger Clough wrote:

1) Yes, numbers float in a sea of universal mind (the One).

2) Here's a thought. If the universe acts like a gigantic
homunculus, with the supreme monad or One as its mind,
then could there be a solipsism to our universe such that
other multiverse versions of oiur universe could not access
(the mind of) ours ? Would this be a problem for multiverse
theories ?


Roger Clough,rclo...@verizon.net  10/31/2012

Dear Roger,

   I think that this idea is exactly wrong. The idea that numbers 
float in a sea of universal mind (the One) makes the explanation an 
infinite regress.


Replace the One by arithmetical truth, and the infinite regress 
disappear.


 Dear Bruno,

Only if arithmetic truth is theory independent, but that ruins your 
result! It truth is theory independent then it is impossible for us to 
be able to know of it. All knowledge is 'theory laden' - as David 
Deutsch explains well.




They reappear *in* arithmetical truth, but have fixed points (some 
provably, some non provably). No problem.


Maybe you might write up an explanation of how arithmetic truth is 
independent of any ability to prove it. That might support your idea of 
arithmetic realism against my claim against it.




That is OK if and only if you allow for the concept of the One to be 
Kaufman and Zuckerman's Quine Atom aka Russell operator, but if not 
it does not work. Why? Because numbers have to be distinguishable 
from to have individual values. The totality of numbers is an 
infinity and thus have the property that their proper parts cannot be 
distinguished from their totality. How does the One accomplish this? 
It seems to me that we have to assume that the One is conscious of 
the numbers and that makes the numbers something different from the 
One for 1) to work and this is no different from what a finite mind 
does. My point here is that a mind cannot be infinite because it 
would be incapable of distinguishing it's self from any of its proper 
parts - making it the ultimate solipsist. Do there exist maps between 
the totality of an infinite set to an improper part? If yes, what are 
their necessary properties?


The One is solipsist, as the one is unique and alone. But I don't see 
why it should be conscious. It might be, but I see no evidence for this.


I agree 100% with you on this.



Bruno


--
Onward!

Stephen


--
You received this message because you are subscribed to the Google Groups 
Everything List group.
To post to this group, send email to everything-list@googlegroups.com.
To unsubscribe from this group, send email to 
everything-list+unsubscr...@googlegroups.com.
For more options, visit this group at 
http://groups.google.com/group/everything-list?hl=en.



Re:On the ontological status of elementary arithmetic

2012-11-01 Thread Stephen P. King

On 11/1/2012 11:39 AM, Bruno Marchal wrote:


Enumerate the programs computing functions fro N to N, (or the 
equivalent notion according to your chosen system). let us call 
those functions:  phi_0, phi_1, phi_2, ...  (the phi_i)

Let B be a fixed bijection from N x N to N. So B(x,y) is a number.

The number u is universal if phi_u(B(x,y)) = phi_x(y). And the 
equality means really that either both phi_u(B(x,y)) and phi_x(y) 
are defined (number) and that they are equal, OR they are both 
undefined.


In phi_u(B(x,y)) = phi_x(y), x is called the program, and y the 
data. u is the computer. u i said to emulate the program (machine, 
...) x on the input y.




   OK, but this does not answer my question. What is the ontological 
level mechanism that distinguishes the u and the x and the y from 
each other?


The one you have chosen above. But let continue to use elementary 
arithmetic, as everyone learn it in school. So the answer is: 
elementary arithmetic.



Dear Bruno,'

If there is no entity to chose the elementary arithmetic, how is it 
chosen or even defined such that there exist arithmetic statements that 
can possibly be true or false? We can assume some special Realm or 
entity does the work of choosing the consistent set of arithmetical 
statements or, as I suggest, we can consider the totality of all 
possible physical worlds as the implementers of arithmetic statements 
and thus their provers. Possible physical worlds, taken as a single 
aggregate, is just as timeless and non-located as the Platonic Realm and 
yet we don't need any special pleading for us to believe in them. ;-)
My thinking here follows the reasoning of Jaakko Hintikka. Are you 
familiar with it? Game theoretic semantics for Proof theory
http://www.hf.uio.no/ifikk/forskning/publikasjoner/tidsskrifter/njpl/vol4no2/gamesem.pdf 



--
Onward!

Stephen

--
You received this message because you are subscribed to the Google Groups 
Everything List group.
To post to this group, send email to everything-list@googlegroups.com.
To unsubscribe from this group, send email to 
everything-list+unsubscr...@googlegroups.com.
For more options, visit this group at 
http://groups.google.com/group/everything-list?hl=en.



Re: Communicability

2012-11-01 Thread Stephen P. King

On 11/1/2012 11:47 AM, Bruno Marchal wrote:


On 01 Nov 2012, at 01:01, Stephen P. King wrote:


Dear Bruno,

   Exactly what do these temporal concepts, such as explain, 
solve, interacting and  emulating, mean in an atemporal 
setting? You are mixing temporal and atemporal ideas. ...


Study a good book in theoretical computer science. You told me that 
you have the book by Matiyazevich. he does explicitly emulate Turing 
machine, which have a quite physical look, with a moving head, and 
obeying instruction is a temporal manner, and yet they can be shown to 
be emulated by a the existence or non existence of solution of 
Diophantine equations.


Dear Bruno,

That book, full of wonderful words and equations, is a physical 
object. That physical object is, in my thinking, an example of an 
implementation of the emulation of a Turing Machine... just as the 
image on my TV of Rainbow Dash and her friends is a physical 
implementation of magical Ponies. You seem to ignore the obvious...




But this is already no more an enigma for many physicists which agree 
that temporality is just an illusion resulting from projection from 
higher dimension.


Those physicists are wrong in their belief. This is argued well in 
this paper http://xxx.lanl.gov/abs/gr-qc/9708055 and in any other 
places. I recall a long chat that I had with Julian Barbor. In it I 
tried to ask him about the computational complexity of implementing his 
'time capsule' and 'best matching' ideas, he seemed to not understand 
what the heck I was talking about and yet bemoaned the very problem at 
length in one of his papers on the idea!


From pg 52 of http://www.platonia.com/barbour_hrp2003.pdf

About ten years ago, I did some computer calculations to find such
configurations with the Macintosh computer I then possessed. I was able to
do exhaustive calculations up to N = 27, which took the computer about three
days. Because the number of combinations that must be checked out grows
exponentially with N, even with a modern supercomputer I doubt that
calculations much beyond N = 50 would be feasible.

BTW, it was reading this paper that opened my eyes to the NP-Hard 
problem of Leibniz' Pre-Established Harmony.





I thought you agree that physics (and thus time) is not primitive.


I agree, physics (and all that it such as particles, forces, 
matter, energy) impels cannot be ontologically primitive. But it must 
exist nonetheless. My challenge is showing how. I start with a notion of 
a property neutral totality of all that exists and consider how from 
that ground two aspects emerge simultaneously, the physical and the 
mental as mutually distinct dual aspects that when added together yield 
back the neutrality. This idea is very similar to Russell Standish's 
Theory of Nothing.



This means that they can and need to be explain from non temporal notion.

Arithmetic is the bloc mindspace.


Is it a Singleton? Can it be exactly represented by a Boolean 
Algebra? I see 'mindspace as one half of the dual aspects.


There is nothing more dynamical than the notion of computations, yet, 
they have been discovered in statical math structure.


Mathematical objects are the epitome of static objects. I think 
that this view of math is blinkered. A description of a dynamic process 
may be static, but the evolutionaly Becoming aspect is still there, just 
hidden. Just as a photograph acts to freeze a moment in time...


This is made possible as the statical sequence 0, 1, 2, 3, ... 
reintroduces a lot of quasi-time notion, and it is explained how some 
of them will play the role of the observable timing of events 
locally, by relative numbers.


This is where you make the mistake. You are assuming that the 
ordering of numbers *is* the dynamic. I claim that the ordering of 
numbers *is a representation* of the dynamic. We should be very careful 
when we identify the map with the territory! I agree that there are 
situations when there is an exact isomorphism between map and territory, 
but that is only in the case of  automorphisms and fixed points.
We can use sequences of relative numbers, surely, but only when the 
conditions to define them occur. We cannot assume that the properties of 
relative numbers exist in the absence of the means to define the 
timing, locality and relations required.



--
Onward!

Stephen


--
You received this message because you are subscribed to the Google Groups 
Everything List group.
To post to this group, send email to everything-list@googlegroups.com.
To unsubscribe from this group, send email to 
everything-list+unsubscr...@googlegroups.com.
For more options, visit this group at 
http://groups.google.com/group/everything-list?hl=en.



Detecting Causality in Complex Ecosystems

2012-11-01 Thread Russell Standish
The distinction between correlation and causality occasionally comes
up in this discussion group, so I thought this paper might be of
interest.

Disclaimer - I haven't read it, but it is published in Science, and
one of the authors (Robert May) I have the utmost respect for.

Let me know if you can't find a non paywalled version. I will probably
be able to get it from my institution's e-library.


- Forwarded message from Complexity Digest Administration 
comdigad...@turing.iimas.unam.mx -



Detecting Causality in Complex Ecosystems

  Identifying causal networks is important for effective policy and management 
recommendations on climate, epidemiology, financial regulation, and much else. 
We introduce a method, based on nonlinear state space reconstruction, that can 
distinguish causality from correlation. It extends to nonseparable weakly 
connected dynamic systems (cases not covered by the current Granger causality 
paradigm). The approach is illustrated both by simple models (where, in 
contrast to the real world, we know the underlying equations/relations and so 
can check the validity of our method) and by application to real ecological 
systems, including the controversial sardine-anchovy-temperature problem.


Detecting Causality in Complex Ecosystems
George Sugihara, Robert May, Hao Ye, Chih-hao Hsieh, Ethan Deyle, Michael 
Fogarty, Stephan Munch

Science 26 October 2012:
Vol. 338 no. 6106 pp. 496-500
http://unam.us4.list-manage2.com/track/click?u=0eb0ac9b4e8565f2967a8304bid=9e44b3450ae=d38efa683e

See it on Scoop.it 
(http://www.scoop.it/t/papers/p/3161484398/detecting-causality-in-complex-ecosystems)
 , via Papers (http://www.scoop.it/t/papers)



-- 


Prof Russell Standish  Phone 0425 253119 (mobile)
Principal, High Performance Coders
Visiting Professor of Mathematics  hpco...@hpcoders.com.au
University of New South Wales  http://www.hpcoders.com.au


-- 
You received this message because you are subscribed to the Google Groups 
Everything List group.
To post to this group, send email to everything-list@googlegroups.com.
To unsubscribe from this group, send email to 
everything-list+unsubscr...@googlegroups.com.
For more options, visit this group at 
http://groups.google.com/group/everything-list?hl=en.



Re: Numbers in the Platonic Realm

2012-11-01 Thread Stephen P. King

On 11/1/2012 12:23 PM, Platonist Guitar Cowboy wrote:


Don't get me started on reductionism! I don't believe in it as I
don't believe in ontologically primitive objects that have
particular properties.


Then I don't see how you can make an ontological bet. You're at the 
table, betting on 24 or whatever, but you won't place your chips.

Hi Cowboy,

Where is the Doctor's Office? I want to make an appointment! Until 
its tech is proven, I am taking Dr. McCoy's stance: 
http://www.youtube.com/watch?v=HxKJyeCRVek


--
Onward!

Stephen

--
You received this message because you are subscribed to the Google Groups 
Everything List group.
To post to this group, send email to everything-list@googlegroups.com.
To unsubscribe from this group, send email to 
everything-list+unsubscr...@googlegroups.com.
For more options, visit this group at 
http://groups.google.com/group/everything-list?hl=en.



Re: Solipsism = 1p

2012-11-01 Thread Stathis Papaioannou
On Mon, Oct 29, 2012 at 3:44 AM, Craig Weinberg whatsons...@gmail.comwrote:

I'm talking about *every experiment* that has been done. There is nothing
 to misunderstand. When I change my mind, through my own thought or though
 some image or suggestion, that change is reflected as a passive consequence
 of the macro-level event. I am not at the mercy of the cellular agendas of
 my brain - I can think about all kinds of things. I can take drugs to
 further impose my high level agenda on low level neurology.


You are at the mercy of the cellular agendas of your brain unless you
believe there is a magical effect of consciousness on matter. How else can
I try to explain this? It appears that you are bamboozled by complex
systems, so that even if each simple interaction is understandable
individually you imagine that something mysterious might be happening if
you can't hold all of the interactions in your mind at once. To eliminate
this difficulty, consider a very simple system that manifests
consciousness. Suppose it has only two components, like two billiard balls.
The components could have whatever special qualities are required for
consciousness. For example, the balls could have evolved naturally as part
of a larger organism. When these balls bounce off each other, consciousness
is implemented. Now, the trajectory of these balls is determined completely
by such factors as their position, mass, velocity, elasticity, air density,
gravitational field, and so on. And as they go about their business
bouncing around, consciousness of a basic kind is generated. As they are
moving towards each other the ball system is thinking of the number 3, but
when they hit and bounce apart it changes its mind and thinks of the number
2. Now, would you say the balls bounced apart because the system decided to
think of the number 2, or would you say the system decided to think of the
number 2 because the balls bounced apart?

The question was about two identical computers, one made in a factory,
 the other assembled with fantastic luck from raw materials moving
 about randomly. Will there be any difference in the functioning or
 consciousness (or lack of it) of the two computers?


 Yes. We have no way of knowing whether the self-assembly is due to luck or
 not, so we have to give it the benefit of the doubt. The computer made in
 the factory is subject to the opposite bias, since we know precisely how it
 was fabricated and that it was made for the purpose of simulating
 consciousness. If asked to choose between a known pathological liar who
 claims to be telling the truth, and someone who has never claimed to be
 telling the truth, all things being equal, we have to give the benefit of
 the doubt to the latter, as we have no reason to expect deceit from them.


You haven't answered the question. The spontaneously formed computer is
*exactly the same* as the manufactured one. I give you what is apparently a
brand new iPhone 5, complete with the inscription Designed by Apple in
California, assembled in China. You turn it on and it searches for a WiFi
network, asks you if you want to set it up as a new phone, asks for your
Apple ID, and eventually the home screen appears with the familiar icons. I
then inform you that this phone was formed spontaneously in a distant
galaxy and arrived on Earth after being ejected by a supernova explosion
billions of years ago. You disassemble it and determine that in every
respect it seems the same as a phone from the factory. Do you still think
that this phone would have different experiences purely because of its
origin?


-- 
Stathis Papaioannou

-- 
You received this message because you are subscribed to the Google Groups 
Everything List group.
To post to this group, send email to everything-list@googlegroups.com.
To unsubscribe from this group, send email to 
everything-list+unsubscr...@googlegroups.com.
For more options, visit this group at 
http://groups.google.com/group/everything-list?hl=en.



Re: Solipsism = 1p

2012-11-01 Thread Craig Weinberg


On Thursday, November 1, 2012 8:43:07 PM UTC-4, stathisp wrote:


 On Mon, Oct 29, 2012 at 3:44 AM, Craig Weinberg 
 whats...@gmail.comjavascript:
  wrote:

 I'm talking about *every experiment* that has been done. There is nothing 
 to misunderstand. When I change my mind, through my own thought or though 
 some image or suggestion, that change is reflected as a passive consequence 
 of the macro-level event. I am not at the mercy of the cellular agendas of 
 my brain - I can think about all kinds of things. I can take drugs to 
 further impose my high level agenda on low level neurology.


 You are at the mercy of the cellular agendas of your brain unless you 
 believe there is a magical effect of consciousness on matter. 


I am at the mercy of the cellular agendas of my brain - absolutely, but the 
cells of my brain are, in some cases, at the mercy of my agenda. If I want 
to stay awake all night playing with some interesting toy, my circadian 
rhythms are going to have to wait, for a while anyways.

 

 How else can I try to explain this?


You have already explained it over and over. You aren't listening to me. I 
understand every bit of your argument. It is my argument that you don't 
understand. I used to believe what you believe. I know better now. You have 
nothing to teach me. Your choices are to listen, not to listen, or bang 
your head against the wall telling me what I already know.
 

 It appears that you are bamboozled by complex systems,


Nope. You are projecting stupidity onto me because your ego can't tolerate 
my disagreement with you.
 

 so that even if each simple interaction is understandable individually you 
 imagine that something mysterious might be happening if you can't hold all 
 of the interactions in your mind at once. To eliminate this difficulty, 
 consider a very simple system that manifests consciousness. Suppose it has 
 only two components, like two billiard balls. The components could have 
 whatever special qualities are required for consciousness. For example, the 
 balls could have evolved naturally as part of a larger organism. When these 
 balls bounce off each other, consciousness is implemented. Now, the 
 trajectory of these balls is determined completely by such factors as their 
 position, mass, velocity, elasticity, air density, gravitational field, and 
 so on. And as they go about their business bouncing around, consciousness 
 of a basic kind is generated. As they are moving towards each other the 
 ball system is thinking of the number 3, but when they hit and bounce apart 
 it changes its mind and thinks of the number 2. Now, would you say the 
 balls bounced apart because the system decided to think of the number 2, or 
 would you say the system decided to think of the number 2 because the balls 
 bounced apart?


The difference between A) Balls bouncing because the system thought of a 
number and B) The system thought of a number because balls bounce is a 
matter of how the system interprets itself. Neither are primitively real. 
Consciousness is the capacity to discern different categories of realism. 
You dramatically underestimate the extent to which consciousness defines 
the universe. It is total.



 The question was about two identical computers, one made in a factory, 
 the other assembled with fantastic luck from raw materials moving 
 about randomly. Will there be any difference in the functioning or 
 consciousness (or lack of it) of the two computers? 


 Yes. We have no way of knowing whether the self-assembly is due to luck 
 or not, so we have to give it the benefit of the doubt. The computer made 
 in the factory is subject to the opposite bias, since we know precisely how 
 it was fabricated and that it was made for the purpose of simulating 
 consciousness. If asked to choose between a known pathological liar who 
 claims to be telling the truth, and someone who has never claimed to be 
 telling the truth, all things being equal, we have to give the benefit of 
 the doubt to the latter, as we have no reason to expect deceit from them.


 You haven't answered the question. The spontaneously formed computer is 
 *exactly the same* as the manufactured one.


You are begging the question. I am saying that it is an ontological 
impossibility. Each event is a particular unrepeatable event in the history 
of the cosmos on some level. 
 

 I give you what is apparently a brand new iPhone 5, complete with 
 the inscription Designed by Apple in California, assembled in China. You 
 turn it on and it searches for a WiFi network, asks you if you want to set 
 it up as a new phone, asks for your Apple ID, and eventually the home 
 screen appears with the familiar icons. I then inform you that this phone 
 was formed spontaneously in a distant galaxy and arrived on Earth after 
 being ejected by a supernova explosion billions of years ago. 
 You disassemble it and determine that in every respect it seems the same as 
 a phone from the 

Re: Solipsism = 1p

2012-11-01 Thread meekerdb

On 11/1/2012 8:19 PM, Craig Weinberg wrote:
You have already explained it over and over. You aren't listening to me. I understand 
every bit of your argument. It is my argument that you don't understand. I used to 
believe what you believe. I know better now. 


The question is how do you know this.  All I've seen are assertions about what computers 
will never be able to do - which is not evidence for much of anything.


Brent

You have nothing to teach me. Your choices are to listen, not to listen, or bang your 
head against the wall telling me what I already know.


--
You received this message because you are subscribed to the Google Groups 
Everything List group.
To post to this group, send email to everything-list@googlegroups.com.
To unsubscribe from this group, send email to 
everything-list+unsubscr...@googlegroups.com.
For more options, visit this group at 
http://groups.google.com/group/everything-list?hl=en.



Re: Solipsism = 1p

2012-11-01 Thread Stathis Papaioannou
On Fri, Nov 2, 2012 at 12:19 PM, Craig Weinberg whatsons...@gmail.comwrote:



 On Thursday, November 1, 2012 8:43:07 PM UTC-4, stathisp wrote:


 On Mon, Oct 29, 2012 at 3:44 AM, Craig Weinberg whats...@gmail.comwrote:

 I'm talking about *every experiment* that has been done. There is nothing
 to misunderstand. When I change my mind, through my own thought or though
 some image or suggestion, that change is reflected as a passive consequence
 of the macro-level event. I am not at the mercy of the cellular agendas of
 my brain - I can think about all kinds of things. I can take drugs to
 further impose my high level agenda on low level neurology.


 You are at the mercy of the cellular agendas of your brain unless you
 believe there is a magical effect of consciousness on matter.


 I am at the mercy of the cellular agendas of my brain - absolutely, but
 the cells of my brain are, in some cases, at the mercy of my agenda. If I
 want to stay awake all night playing with some interesting toy, my
 circadian rhythms are going to have to wait, for a while anyways.


But you can't stay awake unless your hardware allows it. You can't decide
to do anything unless your brain goes into the particular configuration
consistent with that decision, and the movement into that configuration is
determined by physical factors. The experiential aspect of it is completely
invisible to a scientist examining your brain.


 How else can I try to explain this?


 You have already explained it over and over. You aren't listening to me. I
 understand every bit of your argument. It is my argument that you don't
 understand. I used to believe what you believe. I know better now. You have
 nothing to teach me. Your choices are to listen, not to listen, or bang
 your head against the wall telling me what I already know.


 It appears that you are bamboozled by complex systems,


 Nope. You are projecting stupidity onto me because your ego can't tolerate
 my disagreement with you.


It's not stupidity, it's impossible for a normal human to hold in his mind
the entire complex workings of a brain.


  so that even if each simple interaction is understandable individually
 you imagine that something mysterious might be happening if you can't hold
 all of the interactions in your mind at once. To eliminate this difficulty,
 consider a very simple system that manifests consciousness. Suppose it has
 only two components, like two billiard balls. The components could have
 whatever special qualities are required for consciousness. For example, the
 balls could have evolved naturally as part of a larger organism. When these
 balls bounce off each other, consciousness is implemented. Now, the
 trajectory of these balls is determined completely by such factors as their
 position, mass, velocity, elasticity, air density, gravitational field, and
 so on. And as they go about their business bouncing around, consciousness
 of a basic kind is generated. As they are moving towards each other the
 ball system is thinking of the number 3, but when they hit and bounce apart
 it changes its mind and thinks of the number 2. Now, would you say the
 balls bounced apart because the system decided to think of the number 2, or
 would you say the system decided to think of the number 2 because the balls
 bounced apart?


 The difference between A) Balls bouncing because the system thought of a
 number and B) The system thought of a number because balls bounce is a
 matter of how the system interprets itself. Neither are primitively real.
 Consciousness is the capacity to discern different categories of realism.
 You dramatically underestimate the extent to which consciousness defines
 the universe. It is total.


The ball system believes that the bouncing apart happened because of its
decision. That is the nature of conscious systems: even if they are able to
see their own internal workings they still have the feeling I did it
because I wanted to. Which is true, I did do it because I wanted to, but
the wanting, the decision and the action are all caused by the physical
processes.


  The question was about two identical computers, one made in a factory,
 the other assembled with fantastic luck from raw materials moving
 about randomly. Will there be any difference in the functioning or
 consciousness (or lack of it) of the two computers?


 Yes. We have no way of knowing whether the self-assembly is due to luck
 or not, so we have to give it the benefit of the doubt. The computer made
 in the factory is subject to the opposite bias, since we know precisely how
 it was fabricated and that it was made for the purpose of simulating
 consciousness. If asked to choose between a known pathological liar who
 claims to be telling the truth, and someone who has never claimed to be
 telling the truth, all things being equal, we have to give the benefit of
 the doubt to the latter, as we have no reason to expect deceit from them.


 You haven't answered the question. The 

Re: Solipsism = 1p

2012-11-01 Thread Craig Weinberg


On Thursday, November 1, 2012 10:03:18 PM UTC-4, Brent wrote:

 On 11/1/2012 8:19 PM, Craig Weinberg wrote: 
  You have already explained it over and over. You aren't listening to me. 
 I understand 
  every bit of your argument. It is my argument that you don't understand. 
 I used to 
  believe what you believe. I know better now. 

 The question is how do you know this.  All I've seen are assertions about 
 what computers 
 will never be able to do - which is not evidence for much of anything. 


I don't know it, I understand it. My understanding could be incorrect, but 
I have seen no reason to suspect that so far. My purpose is not to make 
assertions about what computers will never be able to do, it is to present 
a framework for the organization of consciousness in the universe. The fact 
that computers thus far are no more sentient than other machines (something 
which should be and would be obvious to anyone not enthralled with science 
fiction religiosity about AI) makes sense in my framework, given that 
qualities of participation and perception accumulate through experience 
itself and cannot be imported from a completely foreign context. My model 
suggests that most physical qualities can only be experienced first hand 
from the inside out, so that a device built entirely on exterior qualities 
(positions in space) has no chance of accidentally reproducing an 
interiority which has developed longitudinally through time as sense 
experience.

Craig


 Brent 

  You have nothing to teach me. Your choices are to listen, not to listen, 
 or bang your 
  head against the wall telling me what I already know. 



-- 
You received this message because you are subscribed to the Google Groups 
Everything List group.
To view this discussion on the web visit 
https://groups.google.com/d/msg/everything-list/-/2RhLxV1Y16oJ.
To post to this group, send email to everything-list@googlegroups.com.
To unsubscribe from this group, send email to 
everything-list+unsubscr...@googlegroups.com.
For more options, visit this group at 
http://groups.google.com/group/everything-list?hl=en.



Re: Life: origin, purpose, and qualia spectrum

2012-11-01 Thread Stephen P. King

On 10/31/2012 9:48 PM, Hal Ruhl wrote:

Hi Everyone:

I would like to restart my participation on the list by having a discussion 
regarding the aspects of what we call “life” in our universe starting in a 
simple manner as follows: [terms not defined herein have the usual “Laws of 
Physics” definition]

1) Definition (1):  Energy (E) is the ability to subject a mass to a force.

2) There are several types of energy currently known:

  a) Mass itself via the conversion: [M = E/(c*c)]
  b) Gravitational
  c) Electromagnetic
  d) Nuclear [Strong and Weak forces]
  e) Dark Energy


Hi Hal,

Nice post! Any way that the energy/force/work relation can be 
considered as a broken symmetry restoration concept?




3) Definition (2) Work (W)  Work is the flow of energy amongst the various 
types by means of a change in the spatial configuration, dynamics and/or amount 
of mass in a system brought about by an actual application of a force to a mass.

4) The exact original distribution of energy amongst the various types can’t be 
reestablished and the new configuration can’t do as much work as the prior 
configuration was capable of doing. [Second Law of Thermodynamics]


Isn't the maximum entropy of a system a type of symmetry, where all 
equiprobable states look the same?




5) Time is not a factor: Once a flow of energy is possible it will take place 
immediately.

6) Conclusion (1):  Since life is an energy flow conduit, wherever the 
possibility of life exists life will appear as rapidly as possible.  The 
“origin” of life herein.


Let me refer you to a very old paper of mine: 
http://webpages.charter.net/stephenk1/Outlaw/life.html




7) Some energy flows are prevented by what are known [in my memory] as “Energy 
Flow Hang-up Barriers” such as nuclear bonding coefficient issues, spatial 
configuration, spin, other spatial dynamics, ignition temperature requirements, 
electromagnetic repulsion, etc.  [“Energy Flow Hang-up Barriers” is not my 
terminology – I think there was a twenty year or so old article in Scientific 
American I am looking for and a quick Internet search found a discussion of the 
repulsion hang-up in “Cosmology The Science of the Universe” by Edward Robert 
Harrison.

8) Once life is present it will immediately punch as many holes in as many 
Energy Hang-up Barriers as the details of the particular life entity involved 
allows – this is how it realizes its energy flow conduit character.  The 
“purpose” of life herein.  In other words life’s purpose is to hasten the heat 
death of its host universe.

9) Now add in evolution which is a random walk with a lower but no upper bound.


Do you see mutation as a one-to-many map and selection as a many 
-to-one map?




A discussion of the possible consequences [such as qualia levels of particular 
life entities - like degrees of consciousness] should await a critique and 
possibly a revision of the above.

Comments are eagerly sought.

Thank you


Nice!

--
Onward!

Stephen


--
You received this message because you are subscribed to the Google Groups 
Everything List group.
To post to this group, send email to everything-list@googlegroups.com.
To unsubscribe from this group, send email to 
everything-list+unsubscr...@googlegroups.com.
For more options, visit this group at 
http://groups.google.com/group/everything-list?hl=en.



Re: Solipsism = 1p

2012-11-01 Thread Craig Weinberg


On Thursday, November 1, 2012 10:03:21 PM UTC-4, stathisp wrote:



 On Fri, Nov 2, 2012 at 12:19 PM, Craig Weinberg 
 whats...@gmail.comjavascript:
  wrote:



 On Thursday, November 1, 2012 8:43:07 PM UTC-4, stathisp wrote:


 On Mon, Oct 29, 2012 at 3:44 AM, Craig Weinberg whats...@gmail.comwrote:

 I'm talking about *every experiment* that has been done. There is 
 nothing to misunderstand. When I change my mind, through my own thought or 
 though some image or suggestion, that change is reflected as a passive 
 consequence of the macro-level event. I am not at the mercy of the 
 cellular 
 agendas of my brain - I can think about all kinds of things. I can take 
 drugs to further impose my high level agenda on low level neurology.


 You are at the mercy of the cellular agendas of your brain unless you 
 believe there is a magical effect of consciousness on matter. 


 I am at the mercy of the cellular agendas of my brain - absolutely, but 
 the cells of my brain are, in some cases, at the mercy of my agenda. If I 
 want to stay awake all night playing with some interesting toy, my 
 circadian rhythms are going to have to wait, for a while anyways.


 But you can't stay awake unless your hardware allows it. 


So what? I can't shoot a gun unless the trigger works. Does that mean I'm 
not shooting the gun by pulling the trigger?
 

 You can't decide to do anything unless your brain goes into the particular 
 configuration consistent with that decision, and the movement into that 
 configuration is determined by physical factors. 


The movement of the molecules of your brain *is* your decision. That's what 
I am telling you but you won't see it. You are only able to see it as a one 
way street which makes no sense. What you are saying is like 'water is ice 
but ice is not water'. If I feel something when something happens in my 
brain, then that means that whatever happens in my brain is also an event 
in the universe when something is felt. That means molecules feel and see. 
You could say that groups of molecules feel and see, and that's ok too, but 
you think it's the 'groupiness' that sees and not the physical reality of 
the molecules themselves. I am saying that there is no independent 
groupiness... it is a fantasy. Incorrect. 

What this means is that molecules as we see them are not the whole story, 
just as the brain and its actions are not the whole story. We are the other 
half of the story and we are not made of neurotransmitters or cells any 
more than a song we make up is our body. Two different ontological schemas. 
Two opposite schemas twisted orthogonally by the private time to public 
space juxtaposition.
 

 The experiential aspect of it is completely invisible to a 
 scientist examining your brain.
  

 How else can I try to explain this?


 You have already explained it over and over. You aren't listening to me. 
 I understand every bit of your argument. It is my argument that you don't 
 understand. I used to believe what you believe. I know better now. You have 
 nothing to teach me. Your choices are to listen, not to listen, or bang 
 your head against the wall telling me what I already know.
  

 It appears that you are bamboozled by complex systems,


 Nope. You are projecting stupidity onto me because your ego can't 
 tolerate my disagreement with you.


 It's not stupidity, it's impossible for a normal human to hold in his mind 
 the entire complex workings of a brain.


Maybe I misunderstood what you meant by  It appears that you are 
bamboozled by complex systems.

 

  so that even if each simple interaction is understandable individually 
 you imagine that something mysterious might be happening if you can't hold 
 all of the interactions in your mind at once. To eliminate this difficulty, 
 consider a very simple system that manifests consciousness. Suppose it has 
 only two components, like two billiard balls. The components could have 
 whatever special qualities are required for consciousness. For example, the 
 balls could have evolved naturally as part of a larger organism. When these 
 balls bounce off each other, consciousness is implemented. Now, the 
 trajectory of these balls is determined completely by such factors as their 
 position, mass, velocity, elasticity, air density, gravitational field, and 
 so on. And as they go about their business bouncing around, consciousness 
 of a basic kind is generated. As they are moving towards each other the 
 ball system is thinking of the number 3, but when they hit and bounce apart 
 it changes its mind and thinks of the number 2. Now, would you say the 
 balls bounced apart because the system decided to think of the number 2, or 
 would you say the system decided to think of the number 2 because the balls 
 bounced apart?


 The difference between A) Balls bouncing because the system thought of a 
 number and B) The system thought of a number because balls bounce is a 
 matter of how the system interprets itself.